Difference between revisions of "2024 AMC 10A Problems/Problem 1"
(→Video Solution by SpreadTheMathLove) |
(→Video Solution by Math from my desk) |
||
Line 1: | Line 1: | ||
{{duplicate|[[2024 AMC 10A Problems/Problem 1|2024 AMC 10A #1]] and [[2024 AMC 12A Problems/Problem 1|2024 AMC 12A #1]]}} | {{duplicate|[[2024 AMC 10A Problems/Problem 1|2024 AMC 10A #1]] and [[2024 AMC 12A Problems/Problem 1|2024 AMC 12A #1]]}} | ||
− | == | + | ==Aura. == |
− | https://www.youtube.com/watch?v= | + | https://www.youtube.com/watch?v=bd5l5NtzoWc |
Revision as of 02:10, 31 January 2025
- The following problem is from both the 2024 AMC 10A #1 and 2024 AMC 12A #1, so both problems redirect to this page.